Оценка ожидаемых значений

Я буду обозначать операторов шляпами. Предположим, мы получили оператор вида я [ п ^ , загар 1 ( е Икс ^ ) ] и мы хотим рассчитать амплитуду перехода из состояния | п я в то же состояние | п я , как это может произойти в задачах упругого рассеяния. Ниже приведены две оценки, которые выглядят правильными, но дают два разных результата. Обозначая ф ^ "=" загар 1 ( е Икс ^ ) ,

1)

п я | я [ п ^ , ф ^ ] | п я "=" я п я | ( п ^ ф ^ ф ^ п ^ ) | п я "=" я п я | ( п я ф ^ ф ^ п я ) | п я "=" я п я п я | ( ф ^ ф ^ ) | п я "=" 0

2)

п я | я [ п ^ , ф ^ ] | п я "=" я п я | ( я ф ^ Икс ) | п я "=" п я | е Икс ^ 1 + е 2 Икс ^ | п я "=" п я | г Икс | Икс Икс | е Икс ^ 1 + е 2 Икс ^ | п я "=" г Икс е Икс 1 + е 2 Икс п я | Икс Икс | п я "=" г Икс е Икс 1 + е 2 Икс е я п я Икс 2 π е я п я Икс 2 π "=" 1 2 π + г Икс е Икс 1 + е 2 Икс "=" 1 2 π π 2 "=" 1 4
Интеграл в оценке 2) можно найти в Jeffrey, Eq. (11), разд. 15.3.1.

Я выбрал форму ф ^ принести желаемые результаты, но вопрос общий. Многие другие более простые формы могут быть использованы для получения аналогичных противоречивых результатов.

Кроме того, результаты можно обобщить, сказав, что: из первой оценки результат всегда равен нулю, из второй оценки результат всегда равен нулю. ф | Икс "=" + ф | Икс "=" .

Что я делаю неправильно?

Простите, про первую строчку в вашем 2), ф ^ Икс "=" А р с Т а н ( Икс ^ ) Икс "=" 1 1 + Икс ^ 2 , Я не могу получить е Икс ^ и т. д.
Извините, была опечатка. я имел в виду А р с Т а н ( е Икс ) . Я исправил сейчас. Спасибо.
Можно также более элегантно обобщить этот противоречивый результат следующим образом. Скажем, у нас есть потенциал В ( Икс ^ ) для которого мы хотим вычислить амплитуду п я | В ( Икс ^ ) | п я . я определяю г ( Икс ^ ) "=" Икс В ( Икс ^ ) . Тогда у меня есть п я | В ( Икс ^ ) | п я "=" п я | Икс г ( Икс ^ ) | п я "=" ( я час ) 1 п я | [ п ^ , г ( Икс ^ ) ] | п я "=" 0 . Это выглядит довольно удивительно.

Ответы (4)

Проблема некорректно поставлена ​​с нуля, потому что | п я не является элементом гильбертова пространства и тем более не принадлежит области п ^ . Та же проблема возникает при рассмотрении ф ^ | п я .

Собственно говоря, строго говоря п я | я [ п ^ , ф ^ ] | п я не существует. С математической точки зрения проблема здесь останавливается, поскольку ex falso quodlibet .

Однако кое-что можно сказать с подходящей интерпретацией п я | я [ п ^ , ф ^ ] | п я . Наивно, но прямолинейно можно дать такую ​​интерпретацию, опуская несущественные знаки и константы,

п я | [ п ^ , ф ^ ] | п я "=" я р е я п я Икс ( г г Икс ф ( Икс ) е я п я Икс ) г Икс я р е я п я Икс ф ( Икс ) г г Икс е я п я Икс г Икс . [ 1 ]
Этот интеграл можно вычислить и получить результат (2). Результат (1) не может быть получен при такой прямой интерпретации формализма, поскольку он опирается на формальную самосопряженность я г г Икс , то есть тождество:
я р ψ ( Икс ) г г Икс г ( Икс ) г Икс "=" я р ( г г Икс ψ ( Икс ) ) г ( Икс ) г Икс [ 2 ]
Это тождество на самом деле выполняется для некоторых классов функций ψ , г (в некоторых случаях также, если ψ , г не принадлежат л 2 ( р ) ). Сравнивая с первым слагаемым в правой части [1], имеем, что должно быть,
г ( Икс ) "=" ф ( Икс ) е я п я Икс ψ ( Икс ) "=" е я п я Икс .
Однако эти функции были исправлены только для того, чтобы сделать их ложными [2]!

Что касается вашего первого абзаца: Естественно | п я принадлежит квантовому пространству п ^ . Я просто определяю это так, говоря п ^ | п я "=" п я | п я . Что касается вашего второго абзаца: такое скалярное произведение может быть определено, поскольку векторы и операторы ket/bra принадлежат одному и тому же ( Икс ^ , п ^ ) квантовое пространство. Тем не менее, мы прямо здесь обсуждаем, как его рассчитать, поэтому его значение все еще не определено в отношении этого поста и его ответов.
Нет, оно не принадлежит гильбертовому пространству теории, | п просто формальный объект, это не л 2 ( р ) элемент. В конце концов, это и есть источник всех подобных проблем, если использовать формальные аргументы и полагать, что это строгие процедуры, в которых нет никакого смысла.
Если вы хотите придать смысл всем этим формальным процедурам, вы должны принять формализм Гельфанда оснащенных гильбертовых пространств. Но опять же, даже при таком подходе обсуждаемый нами объект оказывается неоднозначным. На вопрос нет ответа, потому что на самом деле вопроса вообще нет. Это просто неправильный способ обращаться с формализмом.
Я думаю, что только сейчас понял ваш ответ, после многих месяцев! Вы имели в виду, что состояния, отличные от L^2, не являются физическими, и использование их ожидаемого значения с любым оператором не имеет никакого смысла. Гильбертово пространство, рассматриваемое в квантовой механике, состоит только из физических состояний, которые имеют норму = 1. Состояния с бесконечной нормой не могут рассматриваться как физические состояния и должны использоваться только для расширения физических состояний. Верно? Это звучит разумно, на самом деле. Спасибо.
@Wizzerad Верно!
Спасибо! Итак, позвольте мне теперь пойти дальше. Если на этот раз я принимаю физические состояния как < Икс | п п час у с >= е я п Икс В , где В - объем интегрирования (это просто гауссовские состояния, где дельта 0 ), то у меня всегда будет < п п час у с | В ( Икс ^ ) | п п час у с >=< п п час у с | г ( Икс ^ ) / Икс | п п час у с >∝< п п час у с | [ п ^ , г ( Икс ^ ) ] | п п час у с >= 0 , где г ( Икс ) является примитивом В ( Икс ) , для любого В ( Икс ) . Верно? Это каким-то образом означает, что прямое рассеяние физических состояний всегда равно нулю, что действительно может иметь смысл.
Это тонкий вопрос. Однако непосредственно в позиционном представлении < п | В | п >= В В ( Икс ) г Икс . Если В ( Икс ) "=" г г / г Икс для некоторых г который исчезает при больших | Икс | (обратите внимание, что это не что иное, как другой способ сказать, что В В ( Икс ) г Икс "=" 0 ), то окончательный результат 0 . Однако это зависит от формы В ...

Это не ответ непосредственно на ваш случай, а некоторые соответствующие наблюдения. Тем не менее, это слишком долго, чтобы опубликовать это в комментарии.

Рассматриваем аналогичную ситуацию, (шапки опускаю)

(1) п | [ Икс , п ] | п

Есть два способа вычислить математическое ожидание (1).

Первый,

(2) п | Икс п п Икс | п "=" п ( п | Икс | п п | Икс | п ) "=" 0

Второй,

(3) п | я | п "=" я дельта ( 0 ) "=" я

Есть нестыковка. Проблема в,

(4) п | Икс | п "=" г Икс г Икс п | Икс Икс | Икс | Икс Икс | п "=" Икс г Икс "="
, что плохо определено, если только мы не возьмем главное значение.

Мы можем сделать более консервативный расчет в соответствии с первым подходом,

(5) лим п п п | [ Икс , п ] | п "=" лим п п ( п п ) п | Икс | п "=" я лим п п ( п п ) п дельта ( п п ) "=" я лим п п дельта ( п п ) "=" я дельта ( 0 )

Теперь все последовательно.

Я думаю, что ситуация похожа на ф "=" А р с Т а н ( е Икс ) , хотя интеграл в лим п п подход.

Спасибо. Я прошел мимо этого примера, прежде чем сформулировать свой вопрос. Я должен признать, что я решил это так же, но немного по-другому. Я сделал: 1) лим п п п | [ Икс , п ] | п "=" лим п п п | ( Икс п п Икс ) | п "=" 0 2) лим п п п | [ Икс , п ] | п "=" я лим п п п | п "=" я лим п п 0 "=" 0 Однако результат отличается от вашего, хотя они снова согласуются друг с другом. Проблема в том, что этот подход не спасает меня от более сложных операторов, таких как Икс 2 уже. Вот я и пришел к тому, что выше.
Я думаю, что из соглашения о "нормализации", п | п "=" дельта ( п п ) , поэтому вам нужно взять дельта-функцию.
Когда я говорю лим п п Я полагаю п п . Вот как предел имеет смысл для меня в первую очередь. Граница дельты Дирака лим Икс 0 дельта ( Икс ) равен нулю. Конечно дельта не непрерывна, поэтому ее предел не совпадает с ее значением, но все же. Я делаю это неправильно? Так что, да, мы могли бы написать последний отрывок, как вы предложили. лим п п дельта ( п п ) "=" 0 , но результат будет тот же.
Как простодушный физик, я принимаю лим Икс 0 дельта ( Икс ) "=" дельта ( 0 ) "=" . Какой-нибудь математик может ответить на этот вопрос, извините
М-м, ладно. Здесь мы не согласны. Предел понимается как предельное значение функции, приближающееся к желаемой точке, которое может отличаться от функции в этой точке. Мне кажется, вы просто заменяете лим Икс Икс ф ( Икс ) "=" ф ( Икс ) что является условием непрерывности. Это не наш случай, так как Дельта не непрерывна. Если бы я взял ваше определение предела, то я бы не знал, каков предел ступенчатой ​​функции в 0 (это может быть 0 или 1) или предел функции 1/x в 0 (это может быть - или + ). В этих случаях вы должны определить левый и правый пределы
Японял твою точку зрения. Поскольку дельта-функцию Дирака можно определить как дельта ( Икс ) "=" лим а 0 1 а π е Икс 2 / а 2 , я меняю местами пределы, т.е. лим Икс 0 дельта ( Икс ) "=" лим а 0 лим Икс 0 1 а π е Икс 2 / а 2 "=" дельта ( 0 ) . Я не утверждаю, что это законно, однако... На этом сайте должно быть гораздо больше математиков, чем я...
Я не думаю, что это законно. Это может помочь: math.stackexchange.com/questions/15240/… . Замена пределов безопасна (или безопаснее) только тогда, когда ограничения существуют. Более того, если вы возьмете Mathematica и напишете Limit[DiracDelta[x],x->0] вы получите = 0 . :-)
ХОРОШО! Но физически то, что я пытался сделать, это восстановиться. п | [ Икс , п ] | п "=" я п | п принимая лимит. Нулевой результат подразумевал бы нулевую норму собственного состояния импульса, что, однако, кажется мне странным......................... Я не против повторить , я не математик.

(1) Выглядит неправильно просто потому, что вы не применили операторы к состоянию | п правильно. Операторы действуют справа налево, поэтому должно получиться:

(1) ( п ^ ф ^ ф ^ п ^ ) | п "=" п ^ ф ^ | п ф ^ п ^ | п "=" п ^ ( ф ^ | п ) ф ^ ( п ^ | п )
потому что ф ^ необходимо действовать | п прежде чем действовать п ^ в теме. Первый член слева от второй строки (и без учета констант) действительно
(2) п ^ ( ф ^ | п ) "=" Икс ( ф | п ) "=" ф Икс | п + ф п | п п ф | п
это не то, что вы предполагали.

(2) выглядит правильным, потому что он использует квантовый аналог скобки Пуассона для определения коммутатора:

(3) [ п ^ , ф ^ ] "=" я ( п ^ Икс ^ ф ^ п ^ п ^ п ^ ф ^ Икс ^ ) "=" я ф ^ Икс
потому что ф ^ / п ^ "=" 0 .


РЕДАКТИРОВАТЬ

Правильное определение оператора импульса:

п ^ "=" я Икс
Таким образом, применяя недостающие константы к (2) и вычитая ф п | п член из (1), получаем
я ф Икс
что то же самое, что уравнение (3). Таким образом, нет никакого парадокса, потому что вы неправильно применяли свои операторы.


РЕДАКТИРОВАТЬ 2

Проблема OP заключается в путанице с заказами оператора. Когда операторы действуют на бюстгальтеры, мы должны взять (эрмитов) сопряженный оператор:

п | п ^ ф ^ | п "=" ( п | ф ^ п ^ ) | п ( п | п ^ ) ф ^
при использовании среднего члена будет получено уравнение, аналогичное моему уравнению (2) выше (т. е. кет заменяется бюстгальтером).

Я думаю, что нашел ответ, который, по крайней мере, для меня, является удовлетворительным.

Амплитуды перехода из состояния | а в то же состояние | а имеет смысл, если я имею дело со связанным состоянием, волновая функция которого интегрируема в квадрате. Все проблемы, на которые я указал, не возникнут со связанными (интегрируемыми в квадрат) состояниями. С другой стороны, если я имею дело со свободным состоянием, квантовое число которого непрерывно, мне придется вставить в качестве конечного состояния совершенно другое состояние. Я указывал на проблемы рассеяния, но в задачах рассеяния рассеянное состояние | п ф в общем случае различны по отношению к начальному состоянию, хотя иногда их энергия одинакова.

Также может возникнуть соблазн рассмотреть п я | В ( Икс ^ ) | п я как ожидаемое значение оператора В ( Икс ^ ) на государство | п я . Однако это снова выглядит неправильно, если состояние не является интегрируемым в квадрате, например | п я . На самом деле, предположим, что мы хотим получить математическое ожидание оператора п ^ на государство | п я . Мы, естественно, хотим, чтобы результат был п я . Этого не произойдет, если мы запишем ожидаемое значение как

п я | п ^ | п я "=" п я дельта ( 0 ) "=" +
так как состояния не интегрируемы с квадратом. Лучшее, что мы можем получить, это вставить конечное другое состояние и получить распределение
Д "=" п | п ^ | п я "=" п дельта ( п п я )
так что при интеграции мы получим желаемый результат:
< п >= Д г п "=" п я   ,
где < п > означает здесь «среднее значение для p».

На мой взгляд, при работе с матрицами плотности и трассами в квантовом пространстве (x,p) нужно также делать это с осторожностью, учитывая вышеизложенное.

В общем, сейчас я не вижу больше необходимости рассматривать элементы вида п я | В ( Икс ^ ) | п я , каким бы ни был потенциал, если | п я не интегрируется с квадратом. В теории возмущений, которая мне больше всего нужна, первый член ряда Дайсона читается как п ф | п я и приравнивается к нулю, так как мы предполагаем п ф п я . Если мы этого не допускаем, то все взрывается до бесконечности в нулевом порядке, а этого мы не хотим.

ОТРЕДАКТИРОВАНО :

Чтобы подчеркнуть, что я имею в виду, позвольте мне еще раз рассмотреть мои уравнения с интегрируемыми в квадрат состояниями:

п я | Икс "=" е я п я Икс В , Икс | п я "=" е я п я Икс В ,
где В это интеграция «Объем» в 1D (кто-то может назвать это В 1 / 3 ). Уравнение (1) читается как есть, в то время как уравнение. (2) теперь гласит:
п я | я [ п ^ , ф ^ ] | п я "=" г Икс е Икс 1 + е 2 x e i p i x V e i p i x V = V + d x e x 1 + e 2 x = V π 2 = 0   , ( 1.1 )
и несоответствие исчезло. Состояния в (1.1), вероятно, являются хорошим приближением гауссовского состояния с большой пространственной шириной, которое мы получили бы в обычных экспериментах. Поэтому я ожидаю, что 0 результат, который мы нашли бы. На самом деле, строгий расчет с гауссовскими состояниями (которые интегрируются в квадрате) был бы интересен. Мы могли бы отправить пространственную ширину в большие значения, чтобы увидеть, как ведет себя значение ожидания. Я посмотрю, смогу ли я это сделать в ближайшие дни.

Серия Дайсона напоминает мне, когда мы имеем дело с матрицей S в рассеянии, мы пишем С "=" 1 + я Т исключить п я "=" п ф ..
Да, с гауссовскими состояниями, как определено в Сакураи, уравнение. 1.7.35, все становится равным ∝1/d≈0, где d — пространственная ширина, которая здесь предполагается равной >>1 (чтобы аппроксимировать плоскую волну). Это так, поскольку функция под интегралом стремится к 0 быстро по мере увеличения x. Таким образом, результат оценки обсуждаемого здесь элемента равен 0, как и предполагалось. Это строго и, я думаю, не так много осталось обсуждать. Я вижу, что, наверное, поднял кому-то плохое настроение и прошу прощения за это. Я постараюсь опубликовать несколько более счастливых вопросов в следующий раз. :-) Спасибо всем, кто присоединился к обсуждению.